Re: [obm-l] Iezzi matemetica

2003-03-31 Por tôpico Oswaldo Stanziola



Felipe,
Qual é o seu email?

  - Original Message - 
  From: 
  felipe mendona 
  To: [EMAIL PROTECTED] 
  Sent: Sunday, March 30, 2003 5:45 
PM
  Subject: [obm-l] Iezzi matemetica
  
  
  Ola 
  pessoal...colegas da lista
  
  
   
  Apesar de off topic,a minha pergunta podeajudar aqueles que vao prestar 
  exame pro IME e/ou ITA como eu,ai vai ela: A pouco tempo eu 
  comprei a coleçao do Iezzi de matematica(fundamentos de matematica 
  elementar,volms1 a 10, que é a mais indicada para tais exames) nao acompanhada 
  do manual do professor,que é um livro de respostas c/resoluçao completa dos 
  exercicios da coleçao.Pelo que percebi,o manual nao é vendido 
  individualmente.Como é que eu faço pra adquirir tal manual ???
   
  Ao inves de mandar pra lista,enviem para mim o e-mail,afimde evitar 
  problemas com Nicolau.
  
   
  
  Abraço 
  
  
   
  Felipe Mendonça Vitória-ES.
   
  
  
  MSN Messenger: converse com os seus amigos online. Instale grátis. Clique aqui. Get 2 
  months FREE*. 
  = 
  Instruções para entrar na lista, sair da lista e usar a lista em 
  http://www.mat.puc-rio.br/~nicolau/olimp/obm-l.html O administrador desta 
  lista é 
  <[EMAIL PROTECTED]>= 



[obm-l] Fichas e Células

2003-03-31 Por tôpico Cláudio \(Prática\)



Caros colegas da lista:

O problema original foi proposto pelo 
Dirichlete modificado pelo Alexandre A. da Rocha:

N células são dispostas em círculo. Você coloca 
fichas nas células da seguinte forma:
Coloca a 1a. ficha na célula 1.
Pula 1 célula e coloca a 2a. ficha na célula 
3.
Pula 2 células e coloca a 3a. ficha na célula 
6.
E assim por diante...
Dado N, em que passo você estará colocando a 2a. 
ficha numa célula pela 1a. vez?

Pra começar, imagine um no. grande de células. 

Os primeiros passos seriam os 
seguintes:
Passo Célula
1 
1
2 
3
3 
6
4 10
5 15
621

Repare que no k-ésimo passo você estará colocando a 
ficha na célula k*(k+1)/2.
Isso é fácil de se provar por indução:
Chame de c(m) = célula onde a ficha é colocada no 
m-ésimo passo.
c(1) = 1
Suponha que c(k) = k*(k+1)/2
c(k+1) = c(k) + (k+1) = k*(k+1)/2 + (k+1) = 
(k+1)*(k+2)/2
-

Agora, suponha que existemN 
células.
Faça a seguinte correspondência:
Célula(1) == 1, N + 1, 2N + 1, 3N + 1, 
...
Célula(2) == 2, N + 2, 2N + 2, 3N + 2, 
...
...
Célula(N) == N, 2N, 3N, ...
Ou seja, cada número natural corresponde a uma 
célula e dois números naturais x e y correspondem à mesma célula se e somente se 
x = y (mod N).

Agora o problema é achar o menor valor inteiro 
positivode k para o qual existe um p, com 1 =p = k - 1 e tal 
que c(k) = c(p) (mod N) ==
k*(k+1)/2 = p*(p+1)/2 (mod N) 
==
k^2 + k = p^2 + p (mod N) ==
k^2 - p^2 = p - k (mod N) ==
(k - p)*(k + p) = p - k (mod N) ==
(k - p)*(k + p + 1) = 0 (mod N)

Se N é primo com (k - p) ou N é primo com (k + p + 
1) aanálise fica mais fácil.
O caso em que N não é primo com nenhum dos dois é 
mais complicado.

CASO 1: mdc(k - p,N) 
= 1
Então vale k + p+ 1 = 0(mod 
N).
Assim, o menor k é tal que:
k + p = N - 1 e 1 = p = k-1 
==

N é par == k = N/2 e p = 
(N-2)/2
mdc(k - p,N) = (1,N) = 1 == OK

N é ímpar == k = (N+1)/2 e p = 
(N-3)/2
mdc(k - p,N) = mdc(2,N) = 1 == OK


CASO 2: mdc(k + p + 1,N) = 1
Nesse caso, k - p = 0 (mod N).

k - p não pode ser 0, pois p = k-1. 

Assim, vamos tentar k - p = N

Aqui temos uma série de casos a considerar:


Caso 2.1: 
N = 1 ou N = 2 (mod 3)
k = N+1 e p = 1 ==
mdc(k+p+1,N) = mdc(N+3,N) = mdc(N,3) = 
1== OK

Caso 2.2: 
N = 0 (mod 3) e N = 1, 2, 3 ou 4 (mod 5) == 
k = N+2 e p = 2 ==
mdc(k+p+1, N) = mdc(N+5,N) = mdc(N,5) = 1 == OK

Caso 2.3: 
N = 0 (mod 3*5)e N  0 (mod 7)
k = N+3 e p = 3 ==
mdc(k+p+1,N) = mdc(N+7,N) = mdc(N,7) = 1 == OK

Caso 2.4:
N = 0 (mod 3*5*7) e N  0 (mod 11)
k = N+5 e p = 5 ==
mdc(k+p+1,N) = mdc(N+11,N) = mdc(N,11) = 1 == OK

E por aí vai, um primo de cada vez


Finalmente, temos o
CASO 3: mdc(k - p,N)  1 e mdc(k + p 
+ 1,N)  1.

Mas aí eu acho que a análise fica excessivamente 
complicada (além disso, o meu saco acabou...)


Um abraço,
Claudio.



[obm-l] Dvidas

2003-03-31 Por tôpico Cludio \(Prtica\)
Title: Help



Caros colegas da lista:

Gostaria de receber ajuda sobre os seguintes problemas, 
nos quais eu fiz algum progresso mas no consegui concluir.

PROBLEMA 1:
(problema no. 74 da Eureka no. 15)

"Ache todas as funes f: R -- R (R: conjunto dos 
reais) tais que:
f(x+y) + f(x-y) = 2f(x)cos(y) para todos x, y em 
R."
.

Eu cheguei a uma soluo (descrita abaixo) sob a hiptese 
de que f  diferencivel em toda a reta.

O meu problema agora :
1. Achar todas as funes que no sejam diferenciveis em 
toda a reta mas que satisfaam a relao do problema,
OU
2. Provar que qualquer funo que satisfaz a relao  
diferencivel em toda a reta.

Eu suspeito que a segunda alternativa  verdadeira, mas 
no consegui provar.


PROBLEMA 2:
(problema no. 5 da 2a. Vingana Olmpica)

5)(Guilherme Issao)Existem p,onde p e primo,crianas 
dispostas num bairrocomo um tabuleiro p por p.Ha tambem duas distribuidoras 
de doces,aCledmilson Marmotta e a Estrogonofre's.A Cledmilson Marmotta manda 
umvendedor para cada uma das p linhas horizontais,sendo que o vendedor 
dai-esima linha tem i Kg de doce de jilo e distribui igualmente entre as 
pcrianas. Da mesma forma Estrogonofre's manda um vendedor para cada uma 
dasp linhas verticais,sendo que o vendedor da j-esima linha tem j Kg de doce 
dejaca e distribui igualmente entre as p crianas. De quantas maneiras 
podemosescolher um grupo de crianas desse bairro para roubar-lhes os doces 
de modoque a quantidade de cada tipo de doce roubada seja 
inteira?[6]
O problema pode ser refraseado como:Determinar o 
nmero de subconjuntos de {1, 2, ..., p}x{1, 2, ...,p} cujasoma dos 
elementos (definida da forma usual: (a,b)+(c,d)=(a+c,b+d) ) seja um 
parordenado da forma (mp,np) onde m e n so inteiros.

Eu cheguei a determinar o nmero de subconjuntos de 
{1,2,...,p} cuja soma dos elementos  = 0 (mod p).
Este nmero (2^p - 2)/p + 2 (incluindo o 
subconjunto vazio, cuja soma  0).
No entanto, enumerar conjuntos de pares ordenados est 
sendo bem mais complicado...

ONicolau deu a seguinte dica: para cada N, o nmero 
de subconjuntos de N elementos do produto cartesiano  aproximadamente 
constante, logo, a resposta deve ser aproximadamente igual a (2^(p^2) - 
2)/(p^2)



PROBLEMA 3:
[Ir-1999] - Existe um inteiro positivo que  uma 
potncia de 2, tal quens podemos obter outra potncia de 2 pelo rearranjo 
de seus dgitos?
Nesse eu fiz o seguinte:

Suponhamos que a resposta seja sim e que existam inteiros 
positivos m e n,com m  n tais que 2^m e 2^n tm os mesmos 
dgitos.Ento, pelo critrio de divisibilidade por 9, teremos:2^n = 
2^m (mod 9) ==2^(n-m) = 1 (mod 9) ==n - m  mltiplo de 6 = 
ordem de 2 mod 9 ==n = m + 6 ==2^n = 64*2^m 
==2^n tem mais dgitos do que 2^mMas, por hiptese, 2^m e 2^n 
tm os mesmos dgitos (em ordens diferentes)==2^n e 2^m tm o mesmo 
nmero de dgitos ==contradio ==a resposta  noNo 
entanto, essa soluo no  vlida se a representao decimal de 2^n tiver zeros 
"internos" em sua representao decimal estes zeros podero ser movidos para a 
esquerda na representao de 2^m.Por exemplo, podemos ter:2^n = 
(ab00c0def)e2^m = (000afecbd)Nesse caso, 2^n e 2^m tm os mesmos 
dgitos apesar de 2^n  64*2^m == 
asoluo acima no  vlida

Um abrao,
Claudio.


Soluo do Problema 1 supondo que f  diferencivel em 
toda a reta:

Derivando em relao a x: f'(x+y) + f'(x-y) = 
2f'(x)cos(y)
Derivando em relao a y: f'(x+y) - f'(x-y) = 
-2f(x)sen(y)

Assim, resolvendo para f'(x+y) e f'(x-y): 
f'(x+y) = f'(x)cos(y) - f(x)sen(y) (i)
f'(x-y) = f'(x)cos(y)+ f(x)sen(y)

Fazendo x = 0 em (i): f'(y) = f'(0)cos(y)- 
f(0)sen(y)


Integrando: f(y) = f'(0)sen(y)+ f(0)cos(y) + 
K
Fazendo y = 0: f(0) = f'(0)sen(0) + f(0)cos(0) + K 
== 
f(0)= f(0) + K==
K = 0 ==
f(y) = f'(0)sen(y) + f(0)cos(y).

Ou seja,
f(x) = Asen(x) + Bcos(x), onde A = f'(0) e B = 
f(0).

Usando identidades trigonomtricas elementares eu 
verifiquei que quaisquer que sejam A e B, esta f satisfaz a relao do 
problema.



Re: [obm-l] Numero redondo(correçao)

2003-03-31 Por tôpico Johann Peter Gustav Lejeune Dirichlet
Eo processo paa na ficha n-1.
Johann Peter Gustav Lejeune Dirichlet [EMAIL PROTECTED] wrote:

Desculpe,foi mal...temos n celulas em circulo. 
Carlos Gustavo Tamm de Araujo Moreira [EMAIL PROTECTED] wrote: 
Nao entendi: se as fichas sao colocadas muma fileira infinita indexadapor |N o processo nao para nunca, nao e'? Ou voce esta' colocando as fichasnum circulo ? Nesse caso, com quantos compartimentos ?Abracos,GuguTurma,tenho uma questao que esta me matando!!!Temos uma sequencia de fichasque devemos colocar em celulas assim:coloca a FICHA 1 NUM espaço,e indutivamenteao se colocar a ficha k em seu compartimento,saltamos k compartimentos epassamos a colocar a ficha k+1 na proxima celula.O processo para quandoalgum compartimento contiver duas fichas.Para quais k o processo para?TEA WITH ME THAT I BOOK YOUR FACE--Use o melhor sistema de busca da InternetRadar UOL - http://www.radaaruol.com.br==! ===Instruções para entrar na lista, sair da lista e usar a lista emhttp://www.mat.puc-rio.br/~nicolau/olimp/obm-l.htmlO administrador desta lista é <[EMAIL PROTECTED]>==Instruções para entrar na lista, sair da lista e usar a lista emhttp://www.mat.puc-rio.br/~nicolau/olimp/obm-l.htmlO administrador desta lista é <[EMAIL PROTECTED]>=


Yahoo! Mail O melhor e-mail gratuito da internet: 6MB de espaço, antivírus, acesso POP3, filtro contra spam.Yahoo! Mail 
O melhor e-mail gratuito da internet: 6MB de espaço, antivírus, acesso POP3, filtro contra spam.

Re: [obm-l] fracoes parciais

2003-03-31 Por tôpico Luis Lopes
Sauda,c~oes,

Obrigado Gugu (como vc mesmo se assina),
vou dar uma olhada.

Agora podemos demonstrar a la Euler que
\sum_{n = 1} 1 / (n^2 + 1) = (\pi\coth\pi - 1) / 2.

Sejam
P(z) = 1 + z^2/2 + ... +  z^{2n}/(2n)!e
Q(z) = z + z^3/3! + ... + z^{2n+1}/(2n+1)! .

Observe agora que:

i) grau de P  grau de Q;
ii) Q' = P;
iii) lim P = \cosh z; lim Q = \sinh z
iv) \cosh z / \sinh z = \coth z.
v) Q tem 2n+1 raízes simples
vi) as raízes de \sinh z são ik\pi, k = 0,+-1,+-2,...

Conclua que lim P(z)/Q(z)=\coth z =
1/z + 2z [1/(z^2 + \pi^2) + 1/(z^2 + 4\pi^2) + ]

E coloque z=\pi no resultado acima.

Não é totalmente rigoroso mas é interessante.

[]'s
Luís

-Mensagem Original-
De: Carlos Gustavo Tamm de Araujo Moreira [EMAIL PROTECTED]
Para: [EMAIL PROTECTED]
Enviada em: sexta-feira, 28 de março de 2003 22:47
Assunto: Re: [obm-l] fracoes parciais


   Caro Luis,
   Isso so' vale se o grau de P for menor que n, por exemplo: x/(x-1) nao
e'
 igual a 1/(x-1), como o seu enunciado implicaria...
   Seja R(x)=soma(k=1 ate' n)([P(a_k) / Q'(a_k)] . [1 / x - a_k]).
 R(x) e' uma funcao racional cujo denominador e' o produto para k variando
 entre 1 e n de (x-a_k), ou seja,Q(x). Ao multiplicarmos a soma acima por
 Q(x), obtemos um polinomio de grau menor que n. Vamos calcular o valor
desse
 polinomio em a_k: como Q(a_k) vale 0, todos os termos se anulam exceto o
 termo [P(a_k) / Q'(a_k)] . [1 / x - a_k]. O produto de Q(x) por esse termo
 e' [P(a_k) / Q'(a_k)] . [Q(x) / x - a_k]. Como, pela definicao de
derivada,
 lim(x-a_k)(Q(x)/(x-a_k))=Q'(a_k), que nao e' 0, pois a_k  e' raiz simples
 de Q(x), segue que Q(x).R(x) tende a P(a_k) quando x tende a a_k, para
todo
 k. Isso mostra que Q(x).R(x)=P(x), pois a diferenca entre os dois lados e'
 um polinomio de grau menor que n que se anula nos n pontos
a_1,a_2,...,a_n.
O item ii) e' um corolario imediato do item i).
Abracos,
Gugu



=
Instruções para entrar na lista, sair da lista e usar a lista em
http://www.mat.puc-rio.br/~nicolau/olimp/obm-l.html
O administrador desta lista é [EMAIL PROTECTED]
=


Re: [obm-l] Problema de geometria.

2003-03-31 Por tôpico Johann Peter Gustav Lejeune Dirichlet
Quer mais o que meu?E ai SaldanhaQuem e esse Carlos Tomei?
Talvez nao seja a mais bonita mas foi a soluçao que obtive.Veja...
Seja t=^PBA,BC=1 temos AB/sen (20+t)=AP/sen t.Assim AP=1,AB=sen 80/sen 20 e temos sen 80/sen 20=sen(20+t)/sen t.
sen 80/sen 20=cos 10/sen20
Eassim 2sen10sen(20+t) =sent. Vamos resolver isso!!
cos (10+t)-cos(30+t)=sent e ai cos(10+t)=sen(60-t)+sent e isso da cos(10+t)=2sen30cos(30-t) ou se quiser cos(10-t)=cos(30-t).Analisando as possibilidades da t=10 e ai o angulo pedido seria 80-10=70.E fim
Como voces conhecem tanta geometria cearense
Ass.:Johann
"Nicolau C. Saldanha" [EMAIL PROTECTED] wrote:
Oi lista,Mandaram-me hoje o seguinte problema.Seja ABC um triângulo com AB = AC e ^A = 20 graus.Seja P no lado AC com AP = BC.Calcule o ângulo ^CBP.O meu colega de sala Carlos Tomei já conhecevários problemas parecidos e resolveu.A, B e C são os vértices 1, z^4 e z^5 doeneágono regular formado pelas raízes nonas da unidadeonde z = exp(2 pi i/9). Trace a reta de z^3 a z^8 = Qe chame o ponto de interseção de R.O triângulo ARQ é equilátero pois seus ângulos ^A e ^Qsão claramente iguais a 60 graus.Assim R=P (o ponto pedido no problema) e ^CBP = 70 graus(pois a reta BP é bissetriz de ^APQ).Segue uma figura em attach.[]s, N. ATTACHMENT part 2 image/png Yahoo! Mail 
O melhor e-mail gratuito da internet: 6MB de espaço, antivírus, acesso POP3, filtro contra spam.

[obm-l] Grafos e Casamentos

2003-03-31 Por tôpico peterdirichlet1985
Turma,quem conhece o enunciado e a demonstraçao do Teorema dos Casamentos?Estava
tentando pensar nele ao ver esse problema:

Numa festa ha 18 garotos e 18 garotas.Destas 36 pessoas,4 delas tem 2 amigos
cada,16 tem 3 amigos cada e o resto tem 4 amigos cada.Qual o minimo de casais
amigos diferentes que pode haver na festa?

Nao sei se tem algo a ver mas de qualquer modo tai.


TEA WITH ME THAT I BOOK YOUR FACE


--
Use o melhor sistema de busca da Internet
Radar UOL - http://www.radaruol.com.br



=
Instruções para entrar na lista, sair da lista e usar a lista em
http://www.mat.puc-rio.br/~nicolau/olimp/obm-l.html
O administrador desta lista é [EMAIL PROTECTED]
=


[obm-l] Fraçao continua de e

2003-03-31 Por tôpico peterdirichlet1985
Turma,ces ja viram a fraçao continua de 
e=[2;1,2,1,1,4,1,1,6,1,1,8,...,1,1,2n,...]Comom se demonstra isso?

TEA WITH ME THAT I BOOK YOUR FACE


--
Use o melhor sistema de busca da Internet
Radar UOL - http://www.radaruol.com.br



=
Instruções para entrar na lista, sair da lista e usar a lista em
http://www.mat.puc-rio.br/~nicolau/olimp/obm-l.html
O administrador desta lista é [EMAIL PROTECTED]
=


[obm-l] Problema da Tesoura(O Retorno???) e sqrt(pi)

2003-03-31 Por tôpico peterdirichlet1985
Turma,alguem sabe demonstrar esse teorema estranho que me apareceu na Semana
Olimpica?
Mostre que e possivel recortar um circulo em varios mas finitos pedaços
e rearranjar os pedaços sem falhas de modo a formar um quadrado.Cada corte
deve ser ou um arco de circulo ou um segmento de reta.
Que tal se esse fosse pra Eureka!?

TEA WITH ME THAT I BOOK YOUR FACE


--
Use o melhor sistema de busca da Internet
Radar UOL - http://www.radaruol.com.br



=
Instruções para entrar na lista, sair da lista e usar a lista em
http://www.mat.puc-rio.br/~nicolau/olimp/obm-l.html
O administrador desta lista é [EMAIL PROTECTED]
=


[obm-l] Mais Problemas em Aberto

2003-03-31 Por tôpico Cludio \(Prtica\)
Title: Help



Caros colegas da lista:

Aqui vai mais uma compilao de problemas que foram propostos mas cujas 
solues nunca foram publicadas na lista.

1) Prove, usando geometria e trigonometria bsica (por exemplo, via o 
teorema de Ptolomeu), mas sem usar lgebra (o Nicolau j apresentou uma soluo 
usando nos. complexos)ou identidades trigonomtricas "mandrakes" (como as 
que o Lus Lopes mencionou) que:tan(3*Pi/11) + 4*sin(2*Pi/11) = 
sqrt(11)

*

2)Determine todos os primos da forma 101010.101.

*
3)Determine todos os inteiros positivos que podem ser representados de 
maneira nica sob a forma ( x^2+y)/(xy+1).
*

4) Seja f:NR uma funo tal que f(1)=3 
ef(m+n)+f(m-n)-m+n-1=(f(2m)+f(2n))/2 para todos os inteiros no negativos m 
e n com m=n. 
Determine a expresso de f(m).

*

5) Alguns de topologia geral:

Definamos x como ponto de condensao de um subconjunto E de R^n 
sequalquer vizinhana V de x contiver um nmero incontvel de elementos 
deE (isto , se V inter E no for numervel). Seja P o conjunto dos 
pontosde condensao de E. Mostre que5.1) E  numervel se, e 
somente se, P for vazio ( o que acarretaautomaticamente que E no  
numervel sse P no for vazio)5.2) O conjunto dos elementos de E que no so 
pontos de condensao domesmo (E inter complementar de P)  
numervel5.3) P  perfeito ( fechado e todos seus elementos so pontos 
deacumulao do mesmo). Na realidade, todo elemento de P  ponto 
decondensao do mesmo .5.4)Todo elemento de P  ponto de condensao de 
E inter P5.5) O fecho de E inter P  o prprio P5.6) Todo conjunto 
fechado  dado pela unio disjunta de um conjuntoperfeito com um conjunto 
numervel (podendo ser que um destes conjuntosseja vazio) Este  o Teorema 
de Cantor-BendixonEstas 5 afirmaes valem, na realidade, em qualquer 
espao mtricoseparvelPara demonstrarmos as afirmaes, observemos que 
todo conjunto aberto deR^n pode ser dado por uma unio numervel de bolas 
abertas. A coleodas bolas abertas de centro em elementos com coordenadas 
racionais eraios racionais  uma base numervel de R^n.*

6) a e b so inteiros com mdc(a,b) = 1.
Prove que se existe um inteiro m tal que am + b  primo, ento existe uma 
infinidade de inteiros n para os quais an + b  primo.

*

7) A notria 2a. Vingana Olmpica:

7.1)(Yuri Gomes)Seja ABC um triangulo e k sua circunferencia 
circunscrita.D e o ponto medio do arco BC que nao contem A;E e a 
intersecao da mediatriz de BD com BC,F e a intersecao da paralelaa AB por 
D com AC,G e a intersecao de EF com AB e H a de GD com AC.Mostreque o 
triangulo AGH e isosceles.[3]7.2)(Alex Abreu)Defina a sequenciax(1) 
natural ex(n+1)=1+(x(1)x(2)x(3)...x(n)).Prove que existe um primo p que 
nao divide ninguem da sequencia acima.[4]7.3)(Yuri Gomes)Seja ABC um 
triangulo com BAC=60.Seja A' o simetrico de Aem relaao a BC,D o ponto do 
segmento AC tal que AB=AD e H o ortocentrode ABC.Se b e a bissetriz externa 
do angulo BAC e M e N sao os pontos ondeas retas A'D e CH cortam b 
respectivamente,mostre que AM=AN.[4]7.4)(Telmo Luis)Definimos uma 
represa e sua barreira como um par de conjuntosfinitos A e B de pontos do 
reticulado,sendo A conexo pela relaao de vizinhanadada por |a-b|=1 tais 
que dado um elemento a de A,para qualquer x do reticuladocom |a-x|=1 temos 
que x e um elemento de A ou de B.Dado #B=K ache o maiorvalor que #A pode 
assumir.[5]7.5)(Guilherme Issao)Existem p,onde p e primo,crianas 
dispostas num bairrocomo um tabuleiro p por p.Ha tambem duas distribuidoras 
de doces,a CledmilsonMarmotta e a Estrogonofre's.A Cledmilson Marmotta manda 
um vendedor paracada uma das p linhas horizontais,sendo que o vendedor da 
i-esima linhatem i Kg de doce de jilo e distribui igualmente entre as p 
crianas.Da mesmaforma Estrogonofre's manda um vendedor para cada uma das p 
linhas verticais,sendoque o vendedor da j-esima linha tem j Kg de doce de 
jaca e distribui igualmenteentre as p crianas.De quantas maneiras podemos 
escolher um grupo de crianasdesse bairro para roubar-lhes os doces de modo 
que a quantidade de cadatipo de doce roubada seja 
inteira?[6]7.6)(Alex Abreu)Ache todas as funoes f:R/{0}-R sem 
pontos fixos tais quef(y(f(x)-x))=f(x)/y-f(y)/x para todos os x,y 
nao-nulos.[6]7.7)(Humberto Naves)Seja X um subconjunto de R com m 
elementos positivos.DetermineX tal que maximize o numero de subconjuntos de 
X de mesma soma.[8]
*

Um abrao,
Claudio.



[obm-l] Mais Probls em Aberto II

2003-03-31 Por tôpico Cludio \(Prtica\)
Title: Help



8) Dois jogadores esto jogando em um tabuleiro infinito, que consiste 
de quadradinhos 1x1. O jogador 1 escolhe um quadrado e marca nele um 0. 
Ento o jogador 2 escolhe outro quadrado e marca um X, e assim por diante. 
O jogo termina quando alguns dos jogadores completar em uma linha ou uma 
coluna 5 quadrados consecutivos, marcados por ele. Se nenhum dos jogadores 
conseguir, o jogo acaba empatato. Prove que o jogador 2 pode impedir o 
jogador 1 de vencer.(Israel/95).
*
9)Prove, para todo nmero real positivo x,y,z, a seguinte 
inequao: (xy+yz+zx)*[1/(x+y) + 1/(y+z) + 1/(z+x)]=1/4.
*
10)Resolva o sistema de 
equaes:i)raiz(3x)*[1+1/(x+y)]=2ii)raiz(7y)*[1-1/(x+y)]=4*raiz(2)
*
11) Seja a,b,c,d 4 nmeros reais no negativos que satisfazem a 
condio 2*(ab+ac+ad+bc+bd+cd)+abc+abd+acd+bcd=16.Prove que 
a+b+c+d=2/3*(ab+ac+ad+bc+bd+cd) e determine o caso de 
igualdade.
*
12)Seja m e n inteiros positivos tal que n=m. Prove que 
(2^n)*n!= (m+n)!/(m-n)! =(m+m)^n.
*
13) Seja a,b e c medidas dos lados de um tringulo. Prove que: 
raiz(a+b-c)+raiz(b+c-a)+raiz(c+a-b)=raiz(a)+raiz(b)+raiz(c)
*
14)Demonstrar que para quaisquer valores real e x, y e z  vlida a 
desigualdade4x(x+y)(x+z)(x+y+z)+yz=0
*
15) Se a^(b^c) = b^d , c/d pode ser dado em funo de a e b ?

*

16) Seja a funao f:N*U{0} -N*U{0}dada pelas 
propriedades:(f(2n+1))-(f(2n))=6f(n)+1 e f(2n)=f(n) para todo n 
natural.Ache #{x elemento de N,f(x)2003}. 
(A soluo desse vale um doce - cortesia do Dirichlet!)

*

17) 
a) Ao escrevermos a frao 1/3^2002 como um nmero decimal, obtemos uma 
dzima peridica. Qual o nmero de algarismos da perodo?
b) Existe algum inteiro positivo n tal que 1/3^n  uma dzima peridica 
cujo perodo tem um nmero par de algarismos?

*

Se algum se lembrar de algum problema que eu no mencionei, por favor 
mande-o para a lista.

Um abrao,
Claudio.


[obm-l] triângulo

2003-03-31 Por tôpico Rafael
Oi Pessoal!

Tenho uma que não estou conseguindo:

 Num triângulo ABC, um dos ângulos que a mediana
 AM = m(a) forma com o lado BC é igual ao ângulo que
 esta mesma mediana forma com a bissetriz do ângulo
 A. Demonstrar:
 i. a²= 4bc   ii. m(a) = raiz(2).(c - b)/2

Escrevi algumas semelhanças, lei do seno, lei da
bissetriz interna, mas ainda não consegui nenhum
resultado. Se alguém puder me dar uma dica...

Abraços,

Rafael.


___
Yahoo! Mail
O melhor e-mail gratuito da internet: 6MB de espaço, antivírus, acesso POP3, filtro 
contra spam. 
http://br.mail.yahoo.com/
=
Instruções para entrar na lista, sair da lista e usar a lista em
http://www.mat.puc-rio.br/~nicolau/olimp/obm-l.html
O administrador desta lista é [EMAIL PROTECTED]
=


[obm-l] Alguem sabe aonde tem esse livro?

2003-03-31 Por tôpico peterdirichlet1985
Ola turma da ListaAlguem sabe quanto custa e aonde compro o livro das
Olimpiadas Brasileiras de Matematica 9ª a 15ª?u moro em Sao Paulo mas dependendo
do caso pode ser por Correios.

TEA WITH ME THAT I BOOK YOUR FACE


--
Use o melhor sistema de busca da Internet
Radar UOL - http://www.radaruol.com.br



=
Instruções para entrar na lista, sair da lista e usar a lista em
http://www.mat.puc-rio.br/~nicolau/olimp/obm-l.html
O administrador desta lista é [EMAIL PROTECTED]
=


Re: [obm-l] Grafos e Casamentos

2003-03-31 Por tôpico Cláudio \(Prática\)
Oi, JP:

O enunciado do Teorema dos Casamentos é o seguinte:
Sejam A(1), A(2), ..., A(n) conjuntos tais que a união da quaisquer k deles
(1 = k = n) contém no mínimo k elementos distintos. Então é possível
selecionar n elementos distintos, sendo um de cada conjunto.

A demonstração padrão é por indução completa em n, e trata dois casos
separadamente:
i) Para cada k (1 = k  n), a união de cada k conjuntos contém pelo menos
k+1 elementos;
ii) Existem k (1 = k  n) e k conjuntos tais que a sua união tem
exatamemente k elementos.

Se você quiser, depois eu posso mandar a demonstração.

Um abraço,
Claudio.





- Original Message -
From: [EMAIL PROTECTED]
To: [EMAIL PROTECTED]
Sent: Monday, March 31, 2003 2:23 PM
Subject: [obm-l] Grafos e Casamentos


 Turma,quem conhece o enunciado e a demonstraçao do Teorema dos
Casamentos?Estava
 tentando pensar nele ao ver esse problema:

 Numa festa ha 18 garotos e 18 garotas.Destas 36 pessoas,4 delas tem 2
amigos
 cada,16 tem 3 amigos cada e o resto tem 4 amigos cada.Qual o minimo de
casais
 amigos diferentes que pode haver na festa?

 Nao sei se tem algo a ver mas de qualquer modo tai.


 TEA WITH ME THAT I BOOK YOUR FACE


 --
 Use o melhor sistema de busca da Internet
 Radar UOL - http://www.radaruol.com.br



 =
 Instruções para entrar na lista, sair da lista e usar a lista em
 http://www.mat.puc-rio.br/~nicolau/olimp/obm-l.html
 O administrador desta lista é [EMAIL PROTECTED]
 =

=
Instruções para entrar na lista, sair da lista e usar a lista em
http://www.mat.puc-rio.br/~nicolau/olimp/obm-l.html
O administrador desta lista é [EMAIL PROTECTED]
=


[obm-l] Re: [owner-obm-l@sucuri.mat.puc-rio.br: BOUNCE obm-l@sucuri.mat.puc-rio.br: Message too long (20000 chars)]

2003-03-31 Por tôpico Johann Peter Gustav Lejeune Dirichlet
Praticando o Inutilia Truncat...
"Nicolau C. Saldanha" [EMAIL PROTECTED] wrote:

Longo demais, obviamente por não terem sido podadas as partes velhas...[]s, N.Bem,a original era que temos n celulas e o processo para na ficha n-1 exatamente.Para quais n?
Carlos Gustavo Tamm de Araujo Moreira <[EMAIL PROTECTED]>wrote:Bem, pela interpretacao abaixo, que parece razoavel, o problema e' acharuma solucao de k(k+1)/2-r(r+1)/2=0 (mod n) com 1=rk(k+1)/2-r(r+1)/2=(k-r)(k+r+1)/2. Queremos entao achar dois numeros (k-r ek+r+1) com paridades distintas, cuja diferenca e' pelo menos 3, cujo produto e' multiplo de 2n e cuja soma seja minima. Nesse caso seu produto sera'igual a 2n (senao podemos cortar fatores deles para que seu produto seja 2nfazendo a sua soma diminuir - temos so' que cuidar do caso em que ao cortar esses fatores obtemos dois fatores com diferenca 1, e nesse caso o produto poderia ser igual a 4n no caso otimo - por causa dessas coisas seria mais natural comecar com k=0...), e a maior potencia de 2 que divide 2n dividira' um deles. Devemos escolher uma tal fatoracao de 2n (ou de 4n) de modo que os fatoressejam os mais proximos possiveis. E' claro que a forma da solucao otima depende de n. Se n=2^u, por exemplo, teremos k=2^u e r=2^u-1 ( esse e' ounico caso, alem de n=3, em que k=n), e se n  3 e' um primo impar, devemos ter k=(n+1)/2 e r=(n-3)/2.Por outro lado, se n=(u+2)(u-1)/2 entao k=u e r=1; esse e' o caso em que k=(sqrt(9+8n)-1)/2 e' o menor possivel comparado com n.Abracos,GuguObs: Se n=36=8.9/2, o produto no caso otimo (16.9=144) e' 4n, e nao 2n(nesse caso o melhor que podemos conseguir com produto igual a 2n comdiferenca pelo menos 3 entre os fatores, dado que os fatores devem terparidade diferente e' 24.3=72, 24+3=27  25=16+9). Nesse caso, portanto,k=12 e r=3. Esses fenomenos so podem ocorrer quando n e' da forma k(k+1)/2 (e nem sempre ocorrem nesses casos-Yahoo! Mail 
O melhor e-mail gratuito da internet: 6MB de espaço, antivírus, acesso POP3, filtro contra spam.

Re: [obm-l] Mais Problemas em Aberto

2003-03-31 Por tôpico Johann Peter Gustav Lejeune Dirichlet
Vamos laEssa do f(m),use recursao que nem a Eureka 9.Consegui fazer os de geometria da Vingança e mais nada alem do 2.E so marcar angulo!!!
Cláudio_(Prática) [EMAIL PROTECTED] wrote:




Caros colegas da lista:

Aqui vai mais uma compilação de problemas que foram propostos mas cujas soluções nunca foram publicadas na lista.

1) Prove, usando geometria e trigonometria básica (por exemplo, via o teorema de Ptolomeu), mas sem usar álgebra (o Nicolau já apresentou uma solução usando nos. complexos)ou identidades trigonométricas "mandrakes" (como as que o Luís Lopes mencionou) que:tan(3*Pi/11) + 4*sin(2*Pi/11) = sqrt(11)

*

2)Determine todos os primos da forma 101010.101.

*
3)Determine todos os inteiros positivos que podem ser representados de maneira única sob a forma ( x^2+y)/(xy+1).
*

4) Seja f:NR uma função tal que f(1)=3 ef(m+n)+f(m-n)-m+n-1=(f(2m)+f(2n))/2 para todos os inteiros não negativos m e n com m&=n. 
Determine a expressão de f(m).

*

5) Alguns de topologia geral:

Definamos x como ponto de condensação de um subconjunto E de R^n sequalquer vizinhança V de x contiver um número incontável de elementos deE (isto é, se V inter E não for numerável). Seja P o conjunto dos pontosde condensação de E. Mostre que5.1) E é numerável se, e somente se, P for vazio ( o que acarretaautomaticamente que E não é numerável sse P não for vazio)5.2) O conjunto dos elementos de E que não são pontos de condensação domesmo (E inter complementar de P) é numerável5.3) P é perfeito (é fechado e todos seus elementos são pontos deacumulação do mesmo). Na realidade, todo elemento de P é ponto decondensação do mesmo .5.4)Todo elemento de P é ponto de condensação de E inter P5.5) O fecho de E inter P é o próprio P5.6) Todo conjunto fechado é dado pela união disjunta de um conjuntoperfeito com um conjunto numerável (podendo ser que um destes conjuntosseja vazio) Este é o Teorema de Cantor-BendixonEstas 5 afirmações valem, na realidade, em qualquer espaço métricoseparávelPara demonstrarmos as afirmações, observemos que todo conjunto aberto deR^n pode ser dado por uma união numerável de bolas abertas. A coleçãodas bolas abertas de centro em elementos com coordenadas racionais eraios racionais é uma base numerável de R^n.*

6) a e b são inteiros com mdc(a,b) = 1. 
Prove que se existe um inteiro m tal que am + b é primo, então existe uma infinidade de inteiros n para os quais an + b é primo.

*

7) A notória 2a. Vingança Olímpica:

7.1)(Yuri Gomes)Seja ABC um triangulo e k sua circunferencia circunscrita.D e o ponto medio do arco BC que nao contem A;E e a intersecçao da mediatriz de BD com BC,F e a intersecçao da paralelaa AB por D com AC,G e a intersecçao de EF com AB e H a de GD com AC.Mostreque o triangulo AGH e isosceles.[3]7.2)(Alex Abreu)Defina a sequenciax(1) natural ex(n+1)=1+(x(1)x(2)x(3)...x(n)).Prove que existe um primo p que nao divide ninguem da sequencia acima.[4]7.3)(Yuri Gomes)Seja ABC um triangulo com BAC=60º.Seja A' o simetrico de Aem relaçao a BC,D o ponto do segmento AC tal que AB=AD e H o ortocentrode ABC.Se b e a bissetriz externa do angulo BAC e M e N sao os pontos ondeas retas A'D e CH cortam b respectivamente,mostre que AM=AN.[4]7.4)(Telmo Luis)Definimos uma represa e sua barreira como um par de conjuntosfinitos A e B de pontos do reticulado,sendo A conexo pela relaçao de vizinhançadada por |a-b|=1 tais que dado um elemento a de A,para qualquer x do reticuladocom |a-x|=1 temos que x e um elemento de A ou de B.Dado #B=K ache o maiorvalor que #A pode assumir.[5]7.5)(Guilherme Issao)Existem p²,onde p e primo,crianças dispostas num bairrocomo um tabuleiro p por p.Ha tambem duas distribuidoras de doces,a CledmilsonMarmotta e a Estrogonofre's.A Cledmilson Marmotta manda um vendedor paracada uma das p linhas horizontais,sendo que o vendedor da i-esima linhatem i Kg de doce de jilo e distribui igualmente entre as p crianças.Da mesmaforma Estrogonofre's manda um vendedor para cada uma das p linhas verticais,sendoque o vendedor da j-esima linha tem j Kg de doce de jaca e distribui igualmenteentre as p crianças.De quantas maneiras podemos escolher um grupo de criançasdesse bairro para roubar-lhes os doces de modo que a quantidade de cadatipo de doce roubada seja inteira?[6]7.6)(Alex Abreu)Ache todas as funçoes f:R/{0}-R sem pontos fixos tais quef(y(f(x)-x))=f(x)/y-f(y)/x para todos os x,y nao-nulos.[6]7.7)(Humberto Naves)Seja X um subconjunto de R com m elementos positivos.DetermineX tal que maximize o numero de subconjuntos de X de mesma soma.[8]
*

Um abraço,
Claudio.
Yahoo! Mail 
O melhor e-mail gratuito da internet: 6MB de espaço, antivírus, acesso POP3, filtro contra spam.

Re: [obm-l] Re:_[obm-l]_Fraçao_continua_de_e

2003-03-31 Por tôpico Johann Peter Gustav Lejeune Dirichlet
Valeu Claudio!!!Nem sei como agradecerAss.:Johann
Cláudio_(Prática) [EMAIL PROTECTED] wrote:
Oi, JP:Você acha a demonstração aqui:http://research.microsoft.com/~cohn/Papers/e.pdfUm abraço,Claudio.- Original Message -From: <[EMAIL PROTECTED]>To: <[EMAIL PROTECTED]>Sent: Monday, Marchh 31, 2003 2:26 PMSubject: [obm-l] Fraçao continua de e Turma,ces ja viram a fraçao continua de e=[2;1,2,1,1,4,1,1,6,1,1,8,...,1,1,2n,...]Comom se demonstra isso? TEA WITH ME THAT I BOOK YOUR FACE -- Use o melhor sistema de busca da Internet Radar UOL - http://www.radaruol.com.br = Instruções para entrar na lista, sair da lista e usar a lista em http://www.mat.puc-rio.br/~nicolau/olimp/obm-l.html O administrador desta lista é <[EMAIL PROTECTED]> ==Instruções para entrar na lista, sair da lista e usar a lista emhttp://www.mat.puc-rio.br/~nicolau/olimp/obm-l.htmlO administrador desta lista é <[EMAIL PROTECTED]>=Yahoo! Mail 
O melhor e-mail gratuito da internet: 6MB de espaço, antivírus, acesso POP3, filtro contra spam.

[obm-l] Dica de site-Mathematical Excalibur

2003-03-31 Por tôpico Johann Peter Gustav Lejeune Dirichlet
Bem,a dica de hoje sera sobre uma otima revista na internet que andei vasculhando ha alguns dias.Com voces www.math.ust.hk/excalibur !Esse e o endereço do Mathematical Excalibur!!Para le-la voce precisara de uma interface para .pdf ou .ps(se ce num tem va em www.teorema.mat.br ).NaExcalibur 8 voce tem uma seçao de problemas propostos bem interessante.Talvez eu mande minha soluçao pra la!
Te mais!!Ass.:JohannYahoo! Mail 
O melhor e-mail gratuito da internet: 6MB de espaço, antivírus, acesso POP3, filtro contra spam.

Re: [obm-l] Problema da Tesoura(O Retorno???) e sqrt(pi)

2003-03-31 Por tôpico Cláudio \(Prática\)
Oi, JP:

Eu também já ouvi falar nesse resultado, mas parece que o círculo tem de ser
recortado em 10^50 pedaços, ou algo assim.
De qualquer jeito, se alguém tiver a demonstração, eu gostaria de dar uma
olhada.

Um abraço,
Claudio.

- Original Message -
From: [EMAIL PROTECTED]
To: [EMAIL PROTECTED]
Sent: Monday, March 31, 2003 3:07 PM
Subject: [obm-l] Problema da Tesoura(O Retorno???) e sqrt(pi)


 Turma,alguem sabe demonstrar esse teorema estranho que me apareceu na
Semana
 Olimpica?
 Mostre que e possivel recortar um circulo em varios mas finitos pedaços
 e rearranjar os pedaços sem falhas de modo a formar um quadrado.Cada corte
 deve ser ou um arco de circulo ou um segmento de reta.
 Que tal se esse fosse pra Eureka!?

 TEA WITH ME THAT I BOOK YOUR FACE



=
Instruções para entrar na lista, sair da lista e usar a lista em
http://www.mat.puc-rio.br/~nicolau/olimp/obm-l.html
O administrador desta lista é [EMAIL PROTECTED]
=


Re: [obm-l] Rearranjo Aberto

2003-03-31 Por tôpico Marcio
Title: Help



 Bom, esse eh um problema que eu 
mandei pra lista ha muito tempo (antes de eu ver uma msg sua pela primeira vez, 
acho), mas que eu ainda nao sei fazer:
Sejamvarias seqs de termos positivos (a), 
(b), (c), ...e considere as somas do tipo S = a_1*b_1*c_1*... +a_2*b_2*c_2* ... 
+ ... a_n*b_n*c_n*... onde (a_i) eh uma permutacao da 1a sequencia, (b_i) uma 
permutacao da 2a, e assim por diante.
 Mostre que S  mxima quando as 
sequencias tem a mesma ordenacao.
 O caso com 2 sequencias eh o que 
se conhece como "desigualdade do rearranjo"..Ja nocaso com varias 
sequencias, embora pareca ser bem intuitivo, eu nao consegui nenhum progresso 
nao trivial...

 Note que esse teorema eh bem 
interessante. Por exemplo, ele implica MA = MG em particular...
Basta analisar as sequencias:
(a1,a2,a3,...,an) = 
(a1,a2,a3,...,an)
(a1,a2,a3,...,an) = (a2, a3, 
...,an, a1)
(a1,a2,a3,...,an) = (a3, a4, 
... , a1, a2)
...
(a1,a2,a3,...,an) = (an, a1, 
a2, ... , )

Como as n sequencias do lado esquerdo tem mesma 
ordenacao, tem-se a1^n + ... + an^n = n*a1*a2*...*an ..

 Depois tento fazer alguns 
dessess problemas que voce colocou..
 Abracos,
 Marcio

  - Original Message - 
  From: 
  Cludio (Prtica) 
  To: [EMAIL PROTECTED] 
  Sent: Monday, March 31, 2003 3:38 
PM
  Subject: [obm-l] Mais Probls em Aberto 
  II
  
  8) Dois jogadores esto jogando em um tabuleiro infinito, que 
  consiste de quadradinhos 1x1. O jogador 1 escolhe um quadrado e marca nele 
  um 0. Ento o jogador 2 escolhe outro quadrado e marca um X, e assim por 
  diante. O jogo termina quando alguns dos jogadores completar em uma 
  linha ou uma coluna 5 quadrados consecutivos, marcados por ele. Se nenhum 
  dos jogadores conseguir, o jogo acaba empatato. Prove que o jogador 2 pode 
  impedir o jogador 1 de vencer.(Israel/95).
  *


FW: [obm-l] triângulo

2003-03-31 Por tôpico Claudio Buffara

Oi, Rafael:

 Num triângulo ABC, um dos ângulos que a mediana
 AM = m(a) forma com o lado BC é igual ao ângulo que
 esta mesma mediana forma com a bissetriz do ângulo
 A. Demonstrar:
 i. a²= 4bc   ii. m(a) = raiz(2).(c - b)/2

 Escrevi algumas semelhanças, lei do seno, lei da
 bissetriz interna, mas ainda não consegui nenhum
 resultado. Se alguém puder me dar uma dica...

ABC nao pode ser isosceles, pois nesse caso teriamos AM perpendicular a BC e
coincidente com a bissetriz de A. Assim, suponhamos que AB  AC == c  b.
Seja P = ponto de interseção da bissetriz interna de BAC com o lado BC.

AM eh mediana ==
BM = MC = a/2

AP eh bissetriz interna ==
CP/BP = AC/AB ==
CP/BP = b/c
Como BP + PC = BC = a, teremos:
CP = a*b/(b+c);  BP = a*c/(b+c)

PAM = PMA ==
Triângulo APM é isósceles ==
AP = PM

Levando em conta que BP + PM = BM = a/2, teremos:
a*c/(b+c) + PM = a/2 ==
PM = a*(b-c)/[2*(b+c)] = AP


Agora vamos aplicar o teorema de Stewart:

Primeiro em relacao a bissetriz AP:
BC*(AP^2 + BP*PC) = AC^2*BP + AB^2*PC ==

a*(a^2*(b-c)^2/[4*(b+c)^2] + a^2*b*c/(b+c)^2) =
= b^2*a*c/(b+c) + c^2*a*b/(b+c) ==

(simplificando tudo)
a^2 = 4*b*c

Em seguida, em relacao a mediana AM:
BC*(AM^2 + BM*MC) = AC^2*BM + AB^2*MC ==

a*(m^2 + (a/2)*(a/2)) = b^2*a/2 + c^2*a/2  ==

m^2 + a^2/4 = (b^2 + c^2)/2 ==

m^2 = (2*b^2 + 2*c^2 - a^2)/4 ==

(levando em conta que a^2 = 4*b*c)
m^2 = (2*b^2 + 2*c^2 - 4*b*c)/4 ==

m^2 = (b - c)^2/2 ==

m = (b - c)/raiz(2)


Um abraco,
Claudio.








=
Instruções para entrar na lista, sair da lista e usar a lista em
http://www.mat.puc-rio.br/~nicolau/olimp/obm-l.html
O administrador desta lista é [EMAIL PROTECTED]
=


RE: [obm-l] Problema da Tesoura(O Retorno???) e sqrt(pi)

2003-03-31 Por tôpico João Gilberto Ponciano Pereira
É um problema engraçado... Intuitivamente, parece que não dá. Vamos chamar
de perímetro convexo a soma dos arcos convexos de cada pedaço recortado, e
perímetro côncavo a soma dos arcos côncavos de cada pedaço recortado.

A figura inicial tem um perímetro convexo igual a 2pi*r, e um perímetro
côncavo igual a zero. Cada corte em arco, aumenta o perímetro côncavo e o
perímetro convexo na mesma quantidade. Analogamente, cada colagem, reduzem
os perímetros da mesma forma. A figura final tem os dois perímetros igual a
zero...

Tem alguma coisa errada nisso?

-Original Message-
From: Cláudio (Prática) [mailto:[EMAIL PROTECTED]
Sent: Monday, March 31, 2003 5:40 PM
To: [EMAIL PROTECTED]
Subject: Re: [obm-l] Problema da Tesoura(O Retorno???) e sqrt(pi)


Oi, JP:

Eu também já ouvi falar nesse resultado, mas parece que o círculo tem de ser
recortado em 10^50 pedaços, ou algo assim.
De qualquer jeito, se alguém tiver a demonstração, eu gostaria de dar uma
olhada.

Um abraço,
Claudio.

- Original Message -
From: [EMAIL PROTECTED]
To: [EMAIL PROTECTED]
Sent: Monday, March 31, 2003 3:07 PM
Subject: [obm-l] Problema da Tesoura(O Retorno???) e sqrt(pi)


 Turma,alguem sabe demonstrar esse teorema estranho que me apareceu na
Semana
 Olimpica?
 Mostre que e possivel recortar um circulo em varios mas finitos pedaços
 e rearranjar os pedaços sem falhas de modo a formar um quadrado.Cada corte
 deve ser ou um arco de circulo ou um segmento de reta.
 Que tal se esse fosse pra Eureka!?

 TEA WITH ME THAT I BOOK YOUR FACE



=
Instruções para entrar na lista, sair da lista e usar a lista em
http://www.mat.puc-rio.br/~nicolau/olimp/obm-l.html
O administrador desta lista é [EMAIL PROTECTED]
=
=
Instruções para entrar na lista, sair da lista e usar a lista em
http://www.mat.puc-rio.br/~nicolau/olimp/obm-l.html
O administrador desta lista é [EMAIL PROTECTED]
=


Re: [obm-l] Problema da Tesoura(O Retorno???) e sqrt(pi)

2003-03-31 Por tôpico Salvador Addas Zanata


Nao e sabido nem se os cortes sao feitos em um conjunto mensuravel, quanto
mais como sao esses conjuntos. Veja o livro Unsolved problems in
geometry.


Abraco,

Salvador


On Mon, 31 Mar 2003, Nicolau C. Saldanha wrote:

 On Mon, Mar 31, 2003 at 03:07:34PM -0300, [EMAIL PROTECTED] wrote:
  Turma,alguem sabe demonstrar esse teorema estranho que me apareceu na Semana
  Olimpica?
  Mostre que e possivel recortar um circulo em varios mas finitos pedaços
  e rearranjar os pedaços sem falhas de modo a formar um quadrado.Cada corte
  deve ser ou um arco de circulo ou um segmento de reta.
  Que tal se esse fosse pra Eureka!?
 
 Isto me cheira ao problema da quadratura do círculo, versão século XX.
 O teorema (que não é fácil) é que é possível cortar um quadrado
 em um número finito de peças e juntá-las para formar um disco redondo
 de mesma área. Mas as peças são muito complicadas, não é possível
 resolver o problema se os cortes forem limitados a curvas bem comportadas.
 
 Isso parece o paradoxo de Banach-Tarski: é possível decompor uma bola
 em um número finito de pedaços e juntá-los para formar duas bolas,
 cada uma igual à bola original. O teorema mais geral é que se A e B
 são dois subconjuntos de R^3 limitados e de interior nào vazio então
 é possível recortar A em um número finito de pedaços e juntá-los
 para montar B. Note em particular que não existe preservação de volume;
 em R^2 existe, não é possível recortar uma bola pequena para montar
 uma bola grande.
 
 []s, N.
 =
 Instruções para entrar na lista, sair da lista e usar a lista em
 http://www.mat.puc-rio.br/~nicolau/olimp/obm-l.html
 O administrador desta lista é [EMAIL PROTECTED]
 =
 

=
Instruções para entrar na lista, sair da lista e usar a lista em
http://www.mat.puc-rio.br/~nicolau/olimp/obm-l.html
O administrador desta lista é [EMAIL PROTECTED]
=


[obm-l] AJUDA

2003-03-31 Por tôpico Daniel Pini



OLá, alguem poderia me ajudar?
1-O número de seis algarismos N=abcdef é tal que quando 
multipicamos por 2, 3, 4, 5, 6 obtemos números com os mesmos algarismos 
permutados ciclicamente. A soma dos alg. de N é: R:27 

2- O valor de: 
1992-(1991-(1990-(1989-(...-(3-(2-1))... 
é:
R:996
3-N= 999..999 com k algarismos iguais a 9 então o número 
de algarismos de N^3 que são distintos de 9 é: R:k+1
4-Fatore: 
a^4+b^4-c^4-2a^b^2+4abc^2


Re: [obm-l] Problema da Tesoura(O Retorno???) e sqrt(pi)

2003-03-31 Por tôpico Claudio

- Original Message -
From: Cláudio (Prática) [EMAIL PROTECTED]
To: [EMAIL PROTECTED]
Sent: Monday, March 31, 2003 5:40 PM
Subject: Re: [obm-l] Problema da Tesoura(O Retorno???) e sqrt(pi)


 Oi, JP:

 Eu também já ouvi falar nesse resultado, mas parece que o círculo tem de
ser
 recortado em 10^50 pedaços, ou algo assim.
 De qualquer jeito, se alguém tiver a demonstração, eu gostaria de dar uma
 olhada.

 Um abraço,
 Claudio.

 - Original Message -
 From: [EMAIL PROTECTED]
 To: [EMAIL PROTECTED]
 Sent: Monday, March 31, 2003 3:07 PM
 Subject: [obm-l] Problema da Tesoura(O Retorno???) e sqrt(pi)


  Turma,alguem sabe demonstrar esse teorema estranho que me apareceu na
 Semana
  Olimpica?
  Mostre que e possivel recortar um circulo em varios mas finitos pedaços
  e rearranjar os pedaços sem falhas de modo a formar um quadrado.Cada
corte
  deve ser ou um arco de circulo ou um segmento de reta.
  Que tal se esse fosse pra Eureka!?
 
  TEA WITH ME THAT I BOOK YOUR FACE
 
 

 =
 Instruções para entrar na lista, sair da lista e usar a lista em
 http://www.mat.puc-rio.br/~nicolau/olimp/obm-l.html
 O administrador desta lista é [EMAIL PROTECTED]
 =




=
Instruções para entrar na lista, sair da lista e usar a lista em
http://www.mat.puc-rio.br/~nicolau/olimp/obm-l.html
O administrador desta lista é [EMAIL PROTECTED]
=


[obm-l] Questão interessante

2003-03-31 Por tôpico Renato Lira



 Você é um piloto de um 
helicóptero Apache e avista uma fileira de tanques inimigosem forma de 
combate no vale do rio tigre, logo a frente distante 46km.
Sabe-se que:
a) Você se aproxima obedecendo uma P.A.(Progressão 
Aritmética) de números inteiros.

b) Você pode atacar os tanques inimigos a partir de 
7,5 km de distancia, o que ocorre entre o oitavo e o nono termo da 
P.A.

c) O número de tanques em formação é o sétimo termo 
de uma P.G.(Progressão geométrica) cuja razao é o inverso da razão da 
P.A.

d) O oposto do sexto termo da P.G. é o sêxtuplo do 
inverso do sétimo termo de uma P.H.(Progressão Harmônica) e também igual ao 
inverso do quarto termo desta mesma P.H., cujo primeiro termo vale 
1/145.

Pergunta-se:
 Sabendo-se que seu helicóptero 
pode destruir o numero de tanques dado pelo sétimo termo da P.A., quantos 
tanques em formação restarão?


[obm-l] Indignadamente off-topic

2003-03-31 Por tôpico A. C. Morgado




Por favor, nunca me pea para eu me imaginar pilotando um helicptero Apache
e bombardeando iraquianos!
Morgado

Renato Lira wrote:
  
  
 
  
 

   Voc  um piloto de um  helicptero
Apache e avista uma fileira de tanques inimigosem forma de  combate no vale
do rio tigre, logo a frente distante 46km.
 
  Sabe-se que:
 
  a) Voc se aproxima obedecendo uma P.A.(Progresso
 Aritmtica) de nmeros inteiros.
 
  
 
  b) Voc pode atacar os tanques inimigos
a partir de  7,5 km de distancia, o que ocorre entre o oitavo e o nono termo
da  P.A.
 
  
 
  c) O nmero de tanques em formao  o
stimo termo  de uma P.G.(Progresso geomtrica) cuja razao  o inverso da
razo da  P.A.
 
  
 
  d) O oposto do sexto termo da P.G.  o
sxtuplo do  inverso do stimo termo de uma P.H.(Progresso Harmnica) e
tambm igual ao  inverso do quarto termo desta mesma P.H., cujo primeiro
termo vale  1/145.
 
  
 
  Pergunta-se:
 
   Sabendo-se que seu helicptero  pode
destruir o numero de tanques dado pelo stimo termo da P.A., quantos  tanques
em formao restaro?






Re: [obm-l] Mais Problemas em Aberto

2003-03-31 Por tôpico Domingos Jr.
Title: Help



p, o 7.2 eueo Wendelj 
provamos:
http://www.linux.ime.usp.br/~domingos/problema.ps
http://www.linux.ime.usp.br/~domingos/problema.pdf



[obm-l] Integral

2003-03-31 Por tôpico Márcio Venício Pilar Alcântara
Alguém sabe me dizer como eu calculo a integral indefinida de x^x (x elevado
a x)?
Consegui calcular a derivada de y = x^x como sendo y' = (1 + lnx) . x^x

Aguardo solução de alguém,

 Márcio Venício P. Alcântara
 http://www.marcio.ezdir.net
 [EMAIL PROTECTED]
 Departamento de Sistemas e Controle de Energia (DSCE)
 Faculdade de Engenharia Elétrica e Computação (FEEC)
 UNICAMP - Campinas - SP - Brasil


 =
 Instruções para entrar na lista, sair da lista e usar a lista em
 http://www.mat.puc-rio.br/~nicolau/olimp/obm-l.html
 O administrador desta lista é [EMAIL PROTECTED]
 =

=
Instruções para entrar na lista, sair da lista e usar a lista em
http://www.mat.puc-rio.br/~nicolau/olimp/obm-l.html
O administrador desta lista é [EMAIL PROTECTED]
=


Re: [obm-l] Mais Probls em Aberto II

2003-03-31 Por tôpico Fbio Dias Moreira
-BEGIN PGP SIGNED MESSAGE-
Hash: SHA1

On Monday 31 March 2003 15:38, Cludio (Prtica) wrote:
 8) Dois jogadores esto jogando em um tabuleiro
 infinito, que consiste de quadradinhos 1x1. O jogador 1
 escolhe um quadrado e marca nele um 0. Ento o jogador 2
 escolhe outro quadrado e marca um X, e assim por diante.
 O jogo termina quando alguns dos jogadores completar em
 uma linha ou uma coluna 5 quadrados consecutivos,
 marcados por ele. Se nenhum dos jogadores conseguir, o
 jogo acaba empatato. Prove que o jogador 2 pode impedir
 o jogador 1 de vencer.(Israel/95).
 [...]

8)
Ladrilhe o plano da seguinte maneira:

ABCC
ABDD
EEGH
FFGH

Quando seu oponente jogar em um quadrado, jogue no quadrado de mesma letra. 
Obviamente, qualquer linha ou coluna de 5 contm um domin inteiro, logo no 
pode ser preenchida por marcas de um jogador.

 [...]
 17)
 a) Ao escrevermos a frao 1/3^2002 como um nmero decimal, obtemos uma
 dzima peridica. Qual o nmero de algarismos da perodo? b) Existe algum
 inteiro positivo n tal que 1/3^n  uma dzima peridica cujo perodo tem um
 nmero par de algarismos?
 [...]

17)
a)
O conjunto formados pelos invertveis mdulo 3^2002 que so congruentes a 1 
mdulo 9  invariante por uma multiplicao por 10. Esse conjunto tem 
\phi(3^2002)/\phi(9) = 3^2000 elementos. Seja P o produto de todos os seus 
elementos. Ento

P === 10^(3^2000)*P (mod 3^2002)
10^(3^2000) === 1 (mod 3^2002)

Logo a ordem de 10 (mod 3^2002) divide 3^2000. Como a ordem de 10 (mod 3^2002) 
 o nmero de elementos do menor conjunto invariante por uma multiplicao 
por 10, mas como 10 !== 1 (mod 27), no existem conjuntos com 
\phi(3^2002)/\phi(27) === 3^1999 elementos. Logo a ordem de 10 (mod 3^2002)  
mesmo 3^2000, *logo o perodo de 1/3^2002  3^2000*.

b)
No. Seja K = {x | x  invertvel (mod 3^n) e x === 1 (mod 9)}.  bvio que 
10K = K. Se P  o produto dos elementos de K, ento

P === 10^(#(K))*P (mod 3^n)
10^(#(K)) === 1 (mod 3^n)
ord_3^2002(10) | #(K) = \phi(3^n)/\phi(9) = 3^(n-2)

Mas 2 | ord_3^2002(10) = 2 | 3^(n-2), *absurdo*. Logo 1/3^n sempre tem 
perodo mpar (em particular, seu perodo sempre  uma potncia de trs).

[]s,

- -- 
Fbio ctg \pi Dias Moreira
-BEGIN PGP SIGNATURE-
Version: GnuPG v1.0.6 (GNU/Linux)
Comment: For info see http://www.gnupg.org

iD8DBQE+iNnmalOQFrvzGQoRAopiAKCnIycHoC8alkkUs3Rs40pYdFi3oACcDmo+
aegviRKBOA7fJIQz24jyDWk=
=+m/H
-END PGP SIGNATURE-

=
Instrues para entrar na lista, sair da lista e usar a lista em
http://www.mat.puc-rio.br/~nicolau/olimp/obm-l.html
O administrador desta lista  [EMAIL PROTECTED]
=


[obm-l] e-mail

2003-03-31 Por tôpico felipe mendona


 
 Meu e-mail é [EMAIL PROTECTED]...


 Felipe 
  MSN Messenger: converse com os seus amigos online.  Instale grátis. Clique aqui.  
=
Instruções para entrar na lista, sair da lista e usar a lista em
http://www.mat.puc-rio.br/~nicolau/olimp/obm-l.html
O administrador desta lista é <[EMAIL PROTECTED]>
=


[obm-l] e-mail

2003-03-31 Por tôpico felipe mendona
 
 Meu e-mail é [EMAIL PROTECTED]...


 Felipe 
  MSN Hotmail, o maior webmail do Brasil.  Faça o seu agora.  
=
Instruções para entrar na lista, sair da lista e usar a lista em
http://www.mat.puc-rio.br/~nicolau/olimp/obm-l.html
O administrador desta lista é <[EMAIL PROTECTED]>
=


Re: [obm-l] AJUDA

2003-03-31 Por tôpico A. C. Morgado




2) 1992 - 1991 + 1990 -...+2 - 1
Grupando de 2 em 2 aparece uma soma de 1992/2 = 996 parcelas iguais a 1.
3) 9+ 9*10 +...+9*(10^(k-1)) = 9*[10^k -1]/(10 - 1) = 10^k -1
O cubo vale 10^3k - 3*10^2k+3*10^k - 1
103000
3...1
A subtraao dah
999700 2
O numero de cima tem 3k+1 algarismos. Todos sao iguais a 9, exceto: 1 algarismo
2, 1 algarismo 7 e 2k - k - 1 = k - 1 algarismos 0. A resposta eh k+1.

Daniel Pini wrote:
  
  
 
  
 

  OL, alguem poderia me ajudar?
 
  1-O nmero de seis algarismos N=abcdef  tal que
quando  multipicamos por 2, 3, 4, 5, 6 obtemos nmeros com os mesmos algarismos
 permutados ciclicamente. A soma dos alg. de N : R:27  
 
  2- O valor de: 
 
  1992-(1991-(1990-(1989-(...-(3-(2-1))...  :
 
  R:996
 
  3-N= 999..999 com k algarismos iguais a 9 ento
o nmero  de algarismos de N^3 que so distintos de 9 : R:k+1
 
  4-Fatore:  a^4+b^4-c^4-2a^b^2+4abc^2






Re: [obm-l] AJUDA

2003-03-31 Por tôpico A. C. Morgado




1) 
 a=1 (se a1, 6N nao poderia ter a mesma quantidade de algarismos de N).
 3N = bcdef1 (o 1=a so pode aparecer na ultima posiao no 3N, pois o 5N nao
pode terminar em 1 e os outros sao pares). Logo, N termina em 7, f=7. 
 2N termina em 4, 4N termina em 8 e 6N termina em 2, 5N termina em 5. 
Os algarismos sao 1(inicial), 7(final), 4, 8, 2 e 5 (nao sei em que posioes)
S = 27
Se o problema tem soluao, a soluao eh 27.

Daniel Pini wrote:
  
  
 
  
 

  OL, alguem poderia me ajudar?
 
  1-O nmero de seis algarismos N=abcdef  tal que
quando  multipicamos por 2, 3, 4, 5, 6 obtemos nmeros com os mesmos algarismos
 permutados ciclicamente. A soma dos alg. de N : R:27  
 
  






Re: [obm-l] Indignadamente off-topic

2003-03-31 Por tôpico Ricardo Prins

HAHAHA! Resposta: Nenhum, já que o apache fora destruído por um camponês e seu rifle russo!

From: "A. C. Morgado" <[EMAIL PROTECTED]>
Reply-To: [EMAIL PROTECTED] 
To: [EMAIL PROTECTED] 
Subject: [obm-l] Indignadamente off-topic 
Date: Mon, 31 Mar 2003 20:34:55 -0300 
 
Por favor, nunca me peça para eu me imaginar pilotando um 
helicóptero Apache e bombardeando iraquianos! 
Morgado 
 
Renato Lira wrote: 
 
 Você é um piloto de um helicóptero Apache e avista uma fileira 
de tanques inimigos em forma de combate no vale do rio tigre, logo 
a frente distante 46km. 
Sabe-se que: 
a) Você se aproxima obedecendo uma P.A.(Progressão Aritmética) de 
números inteiros. 
 b) Você pode atacar os tanques inimigos a partir de 7,5 km de 
distancia, o que ocorre entre o oitavo e o nono termo da P.A. 
 c) O número de tanques em formação é o sétimo termo de uma 
P.G.(Progressão geométrica) cuja razao é o inverso da razão da P.A. 
 d) O oposto do sexto termo da P.G. é o sêxtuplo do inverso do 
sétimo termo de uma P.H.(Progressão Harmônica) e também igual ao 
inverso do quarto termo desta mesma P.H., cujo primeiro termo vale 
1/145. 
 Pergunta-se: 
 Sabendo-se que seu helicóptero pode destruir o numero de 
tanques dado pelo sétimo termo da P.A., quantos tanques em formação 
restarão? 
 
 
MSN 8 with e-mail virus protection service:  2 months FREE*
=
Instruções para entrar na lista, sair da lista e usar a lista em
http://www.mat.puc-rio.br/~nicolau/olimp/obm-l.html
O administrador desta lista é <[EMAIL PROTECTED]>
=


[obm-l] sqrt(12a^3 - 3)

2003-03-31 Por tôpico Wagner



Oi para todos!

Deêm uma olhada nesse problema abaixo:

Prove que sea é racional, então sqrt(12a^3 - 3) só é racional sea = 1

André T.


Re: [obm-l] AJUDA

2003-03-31 Por tôpico Fábio Dias Moreira
-BEGIN PGP SIGNED MESSAGE-
Hash: SHA1

On Monday 31 March 2003 22:14, A. C. Morgado wrote:
 1)
 a=1 (se a1, 6N nao poderia ter a mesma quantidade de algarismos de N).
 3N = bcdef1 (o 1=a so pode aparecer na ultima posiçao no 3N, pois o 5N
 nao pode terminar em 1 e os outros sao pares). Logo, N termina em 7, f=7.
 2N termina em 4, 4N termina em 8 e 6N termina em 2, 5N termina em 5.
 Os algarismos sao 1(inicial), 7(final), 4, 8, 2 e 5 (nao sei em que
 posiçoes)
 S = 27
 Se o problema tem soluçao, a soluçao eh 27.
 [...]

142857, o período da expansão decimal de 1/7.

[]s,

- -- 
Fábio ctg \pi Dias Moreira
-BEGIN PGP SIGNATURE-
Version: GnuPG v1.0.6 (GNU/Linux)
Comment: For info see http://www.gnupg.org

iD8DBQE+iOx6alOQFrvzGQoRApDSAKC+uxHrYVw3EUHtg4nvZwLidBGcDACdHemt
/xQKXOXcMJNBCSDKZTeKZKY=
=kpn8
-END PGP SIGNATURE-

=
Instruções para entrar na lista, sair da lista e usar a lista em
http://www.mat.puc-rio.br/~nicolau/olimp/obm-l.html
O administrador desta lista é [EMAIL PROTECTED]
=


Re: [obm-l] Mais Probls em Aberto II

2003-03-31 Por tôpico Wagner
Oi para todos!

Isso tambm  a prova das 2 hipteses que eu sugeri para resolver o problema
(Mas essas hipteses no eram suficientes para chegar na resposta, j que a
resposta poderia ser 3^2000 ou 3^2001)

Andr T.



  17)
  a) Ao escrevermos a frao 1/3^2002 como um nmero decimal, obtemos uma
  dzima peridica. Qual o nmero de algarismos da perodo? b) Existe
algum
  inteiro positivo n tal que 1/3^n  uma dzima peridica cujo perodo tem
um
  nmero par de algarismos?
  [...]

 17)
 a)
 O conjunto formados pelos invertveis mdulo 3^2002 que so congruentes a
1
 mdulo 9  invariante por uma multiplicao por 10. Esse conjunto tem
 \phi(3^2002)/\phi(9) = 3^2000 elementos. Seja P o produto de todos os seus
 elementos. Ento

 P === 10^(3^2000)*P (mod 3^2002)
 10^(3^2000) === 1 (mod 3^2002)

 Logo a ordem de 10 (mod 3^2002) divide 3^2000. Como a ordem de 10 (mod
3^2002)
  o nmero de elementos do menor conjunto invariante por uma multiplicao
 por 10, mas como 10 !== 1 (mod 27), no existem conjuntos com
 \phi(3^2002)/\phi(27) === 3^1999 elementos. Logo a ordem de 10 (mod
3^2002) 
 mesmo 3^2000, *logo o perodo de 1/3^2002  3^2000*.

 b)
 No. Seja K = {x | x  invertvel (mod 3^n) e x === 1 (mod 9)}.  bvio
que
 10K = K. Se P  o produto dos elementos de K, ento

 P === 10^(#(K))*P (mod 3^n)
 10^(#(K)) === 1 (mod 3^n)
 ord_3^2002(10) | #(K) = \phi(3^n)/\phi(9) = 3^(n-2)

 Mas 2 | ord_3^2002(10) = 2 | 3^(n-2), *absurdo*. Logo 1/3^n sempre tem
 perodo mpar (em particular, seu perodo sempre  uma potncia de trs).

 []s,

 - --
 Fbio ctg \pi Dias Moreira
 -BEGIN PGP SIGNATURE-
 Version: GnuPG v1.0.6 (GNU/Linux)
 Comment: For info see http://www.gnupg.org

 iD8DBQE+iNnmalOQFrvzGQoRAopiAKCnIycHoC8alkkUs3Rs40pYdFi3oACcDmo+
 aegviRKBOA7fJIQz24jyDWk=
 =+m/H
 -END PGP SIGNATURE-

 =
 Instrues para entrar na lista, sair da lista e usar a lista em
 http://www.mat.puc-rio.br/~nicolau/olimp/obm-l.html
 O administrador desta lista  [EMAIL PROTECTED]
 =


=
Instrues para entrar na lista, sair da lista e usar a lista em
http://www.mat.puc-rio.br/~nicolau/olimp/obm-l.html
O administrador desta lista  [EMAIL PROTECTED]
=


[obm-l] K-ésimo número da sequência! (A reencarnação)

2003-03-31 Por tôpico Helder Suzuki
Se temos todas frações reduzidas entre 0/1 e 1/1
(inclusive) com denominadores = N e ordenadas, qual a
K-ésima fração em função de N e K?

por exemplo
se N = 3
temos:
(0, 1/3, 1/2, 2/3, 1)
A1 = 0, A2 = 1/3, ..., A5 = 1

Abraços,
Helder Toshiro Suzuki

___
Yahoo! GeoCities
Tudo para criar o seu site: ferramentas fáceis de usar, espaço de sobra e acessórios.
http://br.geocities.yahoo.com/
=
Instruções para entrar na lista, sair da lista e usar a lista em
http://www.mat.puc-rio.br/~nicolau/olimp/obm-l.html
O administrador desta lista é [EMAIL PROTECTED]
=


Re: [obm-l] AJUDA

2003-03-31 Por tôpico Ricardo Prins

1 - O número é 142857(aprox.1/7).2 - erm... esboçando uma solução...


2-1=13-1=24-2=25-2=36-3=3...n-n/2=n/2(para n par)1992-996=996(tá feio isso, eu admito...juro que dou uma solução melhor amanhã!)3 - n^3 tem 3n algarismos... Ex.: ^3=99970002observe o padrão que esses numeros formam... tente o mesmo para 999^3 e 9^3 e vc verá do que estou falando. logo, a resposta é n+1.4: é realmente 2a^b^2?
From: "Daniel Pini" <[EMAIL PROTECTED]>
Reply-To: [EMAIL PROTECTED] 
To: <[EMAIL PROTECTED]>
Subject: [obm-l] AJUDA 
Date: Mon, 31 Mar 2003 20:54:07 -0300 
 
OLá, alguem poderia me ajudar? 
1-O número de seis algarismos N=abcdef é tal que quando multipicamos por 2, 3, 4, 5, 6 obtemos números com os mesmos algarismos permutados ciclicamente. A soma dos alg. de N é: R:27 
2- O valor de: 
1992-(1991-(1990-(1989-(...-(3-(2-1))... é: 
R:996 
3-N= 999..999 com k algarismos iguais a 9 então o número de algarismos de N^3 que são distintos de 9 é: R:k+1 
4-Fatore: a^4+b^4-c^4-2a^b^2+4abc^2 
STOP MORE SPAM with the new MSN 8 and get 2 months FREE*
=
Instruções para entrar na lista, sair da lista e usar a lista em
http://www.mat.puc-rio.br/~nicolau/olimp/obm-l.html
O administrador desta lista é <[EMAIL PROTECTED]>
=


[obm-l] Rearranjo generalizado

2003-03-31 Por tôpico Claudio Buffara
Oi, Marcio:

Sobre esse seu problema:

Sejam varias seqs de termos positivos (a), (b), (c), ...e considere as somas
do tipo S = a_1*b_1*c_1*... +a_2*b_2*c_2* ... + ... a_n*b_n*c_n*... onde
(a_i) eh uma permutacao da 1a sequencia, (b_i) uma permutacao da 2a, e assim
por diante.
Mostre que S é máxima quando as sequencias tem a mesma ordenacao.
O caso com 2 sequencias eh o que se conhece como desigualdade do
rearranjo

---

Note que esse teorema eh bem interessante. Por exemplo, ele implica MA
= MG em particular...
Basta analisar as sequencias:
(a1,a2,a3,...,an)   =  (a1,a2,a3,...,an)
(a1,a2,a3,...,an)   =  (a2, a3, ...,an, a1)
(a1,a2,a3,...,an)   =  (a3, a4, ... , a1, a2)
...
(a1,a2,a3,...,an)   =  (an, a1, a2, ...  ,   )
 
Como as n sequencias do lado esquerdo tem mesma ordenacao, tem-se a1^n + ...
+ an^n = n*a1*a2*...*an ..



Eu pensei em usar inducao sobre o numero M de sequencias (M = 2):
 
O caso base (M = 2) eh, como voce disse, a desigualdade do rearranjo.

Supondo que o resultado seja verdadeiro para quaisquer M-1 sequencias (M =
3) de termos positivos, consideremos as M sequencias (A_i), (B_i), (C_i),
..., (Z_i) (achei melhor usar esta notacao do que dois indices) de termos
positivos e as somas correspondentes do tipo:
S = A_1*B_1*...*Z_1 + ... + A_n*B_n*...*Z_n

Inicialmente, aplicamos a hipotese de inducao as M-1 sequencias (A_i*B_i),
(C_i), ..., (Z_i) e concluimos que S eh maxima quando todas estas as
sequencias tem a mesma ordenacao, digamos:
0  A_1*B_1 = ... = A_n*B_n,
0  C_1 = ... = C_n,
...
0  Z_1 = ... = Z_n.

Agora, aplicamos a h.i. as M-1 sequencias (A_i), (B_i*C_i), ..., (Z_i) e
concluimos que S eh maxima quando:
0  A_1 = ... = A_n,
0  B_1*C_1 = ... = B_n*C_n,
...
0  Z_1 = ... = Z_n.

Finalmente, aplicamos a h.i. as M-1 sequencias (A_i*C_i), (B_i), ..., (Z_i)
e concluimos que S eh maxima quando:
0  A-1*C_1 = ... = A_n*C_n,
0  B_1 = ... = B_n,
...
0  Z_1 = ... = Z_n.

Naturalmente, o valor maximo de S serah o mesmo em cada um dos tres casos
acima.

Estas tres aplicacoes da h.i. implicam que S eh maxima quando:
0  A_1 = ... = A_n,
0  B_1 = ... = B_n,
0  C_1 = ... = C_n,
...
0  Z_1 = ... = Z_n,
ou seja, quando as M sequencias tiverem a a mesma ordenacao.

Voce ve algum furo neste raciocinio?

Um abraco,
Claudio. 


=
Instruções para entrar na lista, sair da lista e usar a lista em
http://www.mat.puc-rio.br/~nicolau/olimp/obm-l.html
O administrador desta lista é [EMAIL PROTECTED]
=


[obm-l] Somando reversos até palíndromos

2003-03-31 Por tôpico Helder Suzuki
Se pegamos um número inteiro positivo qualquer e
somamos a ele o seu reverso, pegamos o resultado e o
somamos ao reverso do resultado e assim
sucessivamente, chegaremos sempre a um palíndromo?
É possível provar se sim ou se não?

por exemplo:
Se começarmos com 195:

195 + 591 = 786
786 + 687 = 1473
1473 + 3741 = 5214
5214 + 4125 = 9339

9339 é palíndromo! :)

Se pegarmos o número 59:

59 + 95 = 140 + 14 = 154
154 + 451 = 605
605 + 506 = 

 palíndromo!

(tentem fazer essa operação com outros números, tentem
depois com 196)

Abraços,
Helder Toshiro Suzuki



___
Yahoo! Mail
O melhor e-mail gratuito da internet: 6MB de espaço, antivírus, acesso POP3, filtro 
contra spam. 
http://br.mail.yahoo.com/
=
Instruções para entrar na lista, sair da lista e usar a lista em
http://www.mat.puc-rio.br/~nicolau/olimp/obm-l.html
O administrador desta lista é [EMAIL PROTECTED]
=


Re: [obm-l] Somando reversos até palíndromos

2003-03-31 Por tôpico Fábio Dias Moreira
-BEGIN PGP SIGNED MESSAGE-
Hash: SHA1

On Monday 31 March 2003 23:39, Helder Suzuki wrote:
 Se pegamos um número inteiro positivo qualquer e
 somamos a ele o seu reverso, pegamos o resultado e o
 somamos ao reverso do resultado e assim
 sucessivamente, chegaremos sempre a um palíndromo?
 É possível provar se sim ou se não?
 [...]

Ninguém sabe. Duas referências legais são

http://www.jasondoucette.com/worldrecords.html
http://www.research.att.com/cgi-bin/access.cgi/as/njas/sequences/eisA.cgi?Anum=023108

mas se você quiser saber mais, pesquise um pouco no Google.

[]s,

- -- 
Fábio ctg \pi Dias Moreira
-BEGIN PGP SIGNATURE-
Version: GnuPG v1.0.6 (GNU/Linux)
Comment: For info see http://www.gnupg.org

iD8DBQE+iP8dalOQFrvzGQoRAj9QAJ9Y1ZCsOGA7fRWJT8aoE60Fh5FaPwCgu+09
21voAjeNK/2yXTrC6pKBObg=
=h4d2
-END PGP SIGNATURE-

=
Instruções para entrar na lista, sair da lista e usar a lista em
http://www.mat.puc-rio.br/~nicolau/olimp/obm-l.html
O administrador desta lista é [EMAIL PROTECTED]
=


Re: [obm-l] AJUDA

2003-03-31 Por tôpico Claudio Buffara
Title: Re: [obm-l] AJUDA



Uma curiosidade: o numero desejado eh justamente aquele formado pelos 6 algarismos do periodo de 1/7 quando expresso em decimal.

1/7 = 0,142857 142857 1428

Logo, N = 142857 e SD(N) = 1+4+2+8+5+7 = 27.

on 31.03.03 22:14, A. C. Morgado at [EMAIL PROTECTED] wrote:

1) 
a=1 (se a1, 6N nao poderia ter a mesma quantidade de algarismos de N).
3N = bcdef1 (o 1=a so pode aparecer na ultima posiçao no 3N, pois o 5N nao pode terminar em 1 e os outros sao pares). Logo, N termina em 7, f=7. 
2N termina em 4, 4N termina em 8 e 6N termina em 2, 5N termina em 5. 
Os algarismos sao 1(inicial), 7(final), 4, 8, 2 e 5 (nao sei em que posiçoes)
S = 27
Se o problema tem soluçao, a soluçao eh 27.

Daniel Pini wrote:
 
OLá, alguem poderia me ajudar?
1-O número de seis algarismos N=abcdef é tal que quando multipicamos por 2, 3, 4, 5, 6 obtemos números com os mesmos algarismos permutados ciclicamente. A soma dos alg. de N é: R:27 








[obm-l] Mais Problemas em Aberto - Topologia

2003-03-31 Por tôpico Artur Costa Steiner
 Caros colegas da lista:
 
 Aqui vai mais uma compilação de problemas que foram propostos mas
cujas  soluções nunca foram publicadas na lista.

[Artur Costa Steiner] 
Sobre, Topologia, para os que curtem, aqui vão algumas soluções:
 
5) Alguns de topologia geral:
 
Definamos x como ponto de condensação de um subconjunto E de R^n se
qualquer vizinhança V de x contiver um número incontável de elementos de
E (isto é, se V inter E não for numerável). Seja P o conjunto dos pontos
de condensação de E. Mostre que

5.1) E é numerável se, e somente se, P for vazio ( o que acarreta
automaticamente que E não é numerável sse P não for vazio)

Sabemos que R^n possui uma base numerável, como, por exemplo, a coleção
das  bolas abertas de raios racionais e centros em elementos de
coordenadas racionais. Seja  B = {B_n} esta coleção. Para maior clareza,
provaremos primeiro o item 5.2
 
5.2) O conjunto dos elementos de E que não são pontos de condensação do
mesmo (E inter complementar de P) é numerável.
Definamos W como a união de todos os conjuntos básicos B_n cujas
intercessões com E sejam numeráveis. Seja cP o complementar de P. Vamos
mostrar que W = cP. Se x pertence a W, então x possui uma vizinhança
básica B_n cuja intercessão com E é numerável. Da definição de ponto de
condensação, segue-se que x não é um de tais pontos e que portanto, x
pertence a cP. Se, por outro lado, x pertence a cP, então x possui uma
vizinhança, logo uma vizinhança básica, cuja intercessão com E é
numerável. Da definição de W, segue-se que x pertence a W. Concluimos
assim que W está contido em cP e vice-versa. Logo W = cP. 
Desta conclusão, segue-se agora que E inter cP = E inter W = (E inter
B_1) U (E inter B_2) U(E inter B_n). Como cada (E inter B_n) é
numerável, vemos que E inter cP é dado por uma união numerável de
conjuntos numeráveis. Logo E inter cP é numerável, o que prova 5.2. 
Voltando-se a 5.1, observamos que E = (E inter P) U (E inter cP). Se P
for vazio (isto é, se E não possuir pontos de condensação) então E = E
inter cP, equação que, em virtude do que acabamos de ver, mostra-nos que
E é numerável. Se, por outro lado, P não for vazio, então E possui um
ponto de condensação x e qualquer vizinhança V de x é tal que V inter E
não é numerável. Dado que V inter E é um subconjunto de E, segue-se que
E não é numerável. Isto prova 5.1. Tomando-se as contrapositivas de tais
conclusões, constatamos imediatamente que E não é numerável sse P não
for vazio.

5.3) P é perfeito (é fechado e todos seus elementos são pontos de
acumulação do mesmo). Na realidade, todo elemento de P é ponto de
condensação do mesmo.
Vimos que cP = W é dado por uma união de conjuntos abertos. Logo, cP é
aberto e P é fechado. Alternativamente, podemos chegar a esta mesma
conclusão observando que, se x pertence a cP, então x possui uma
vizinhança V cuja intercessão com E é numerável. Como V é vizinhança de
todos os seus elementos, segue-se que igual condição vale para todo
elemento de V, o que nos mostra que V está contida em cP. Todo elemento
de cP é portanto ponto interior do mesmo, do que deduzimos que cP é
aberto e P é fechado.
Sejam agora p pertencente a P e V uma vizinhaça qualquer de p. Temos a
seguinte equação: V inter E = (V inter E inter P) U (V inter E inter
cP). Pela definição de ponto de condensação, V inter E não é numerável
e, conforme já vimos, E inter cP é numerável. Logo, V inter E inter cP é
numerável, pois é subconjunto de E inter cP. Para que a equação citada
possa vigorar, temos então, necessariamente, que V inter E inter P não
pode ser numerável. Como V é arbitrária, concluimos que p é ponto de
condensação de E inter P e, consequentemente, do próprio P. E como todo
ponto de condensação de um conjunto é, automaticamente, ponto de
acumulação do mesmo, concluimos que todo elemento de P é ponto de
acumulação do mesmo. Logo P é perfeito. OBS. Nesta demonstração
admitimos que P não é vazio. Se P for vazio, então P é trivialmente
perfeito. 
  
5.4)Todo elemento de P é ponto de condensação de E inter P
Conseqüência imediata da demonstração de 5.3. Como corolário, segue-se
que, se P não for vazio, então E inter P não é numerável. Como outro
corolário, temos que todo elemento de E inter P é ponto de condensação
do mesmo.

5.5) O fecho de E inter P é o próprio P
Se x pertence a fecho de E inter P, então então toda vizinhança V de x
intercepta E inter P e, portanto, intercepta P. Logo, V contém um ponto
de condensação de E, o que acarreta que V inter E não seja numerável.
Segue-se que x é ponto de condensação de E e, face a isto, x pertence a
P. Se, por outro lado, x pertence a P, então, conforme vimos, x é ponto
de condensação de E inter P. É então imediato que x pertence ao fecho de
E inter P. Isto prova 5.5
 
5.6) Todo conjunto fechado é dado pela união disjunta de um conjunto
perfeito com um conjunto numerável (podendo ser que um destes conjuntos
seja vazio). Este é o Teorema de Cantor-Bendixon.
Suponhamos que E seja fechado. Temos que E = (E 

Re: [obm-l] K-ésimo número da sequência! (A reencarnação)

2003-03-31 Por tôpico Claudio Buffara
Essa eh a chamada sequencia de Farey de ordem N  (F(N)).

a/b pertence a F(N) == 0 = a = b = N  e  mdc(a,b) = 1.

Alem disso, se o k-esimo termo eh a/b e o (k+1)-esimo eh c/d entao:
a/b  c/d  e  bc - ad = 1, ou seja:
c/d = a/b + 1/(bd).

Apesar de nao fornecer uma formula, o algoritmo abaixo (descrito em An
Introduction to the Theory of Numbers - Hardy-Wright - sec. 3.4) permite
determinar o c/d conhecendo-se a/b:

Como mdc(a,b) =1, existem inteiros x e y tais que bx - ay = 1.

Se (p,q) eh uma solucao particular dessa equacao diofantina (ou seja, bp -
aq = 1) entao, a solucao geral serah:
x = p + a*k
y = q + b*k  (k em Z)

Podemos escolher k de modo que N - b  q + b*k = y = N.

Dessa forma, teremos uma solucao (x,y) tal que:
mdc(x,y) = 1  e  N - b  y = N ==
x/y pertence a F(N)  e  x/y = a/b + 1/(by)  a/b.

Vamos provar, por absurdo, que x/y = c/d.

Suponhamos que x/y  c/d. Entao, x/y  c/d ==
x/y - c/d = (dx - cy)/(dy) = 1/(dy)

Por outro lado,
c/d - a/b = (bc - ad)/(bd) = 1/(bd)

Assim:
1/(by) = (bx - ay)/(by) = x/y - a/b = 1/(dy) + 1/(bd) =
= (b + y)/(bdy)  N/(bdy) = 1/(by) ==
contradicao ==
x/y = c/d

Um abraco,
Claudio. 

on 31.03.03 23:16, Helder Suzuki at [EMAIL PROTECTED] wrote:

 Se temos todas frações reduzidas entre 0/1 e 1/1
 (inclusive) com denominadores = N e ordenadas, qual a
 K-ésima fração em função de N e K?
 
 por exemplo
 se N = 3
 temos:
 (0, 1/3, 1/2, 2/3, 1)
 A1 = 0, A2 = 1/3, ..., A5 = 1
 
 Abraços,
 Helder Toshiro Suzuki
 
 ___
 Yahoo! GeoCities
 Tudo para criar o seu site: ferramentas fáceis de usar, espaço de sobra e
 acessórios.
 http://br.geocities.yahoo.com/
 =
 Instruções para entrar na lista, sair da lista e usar a lista em
 http://www.mat.puc-rio.br/~nicolau/olimp/obm-l.html
 O administrador desta lista é [EMAIL PROTECTED]
 =
 

=
Instruções para entrar na lista, sair da lista e usar a lista em
http://www.mat.puc-rio.br/~nicolau/olimp/obm-l.html
O administrador desta lista é [EMAIL PROTECTED]
=


Re: [obm-l] Problema da Tesoura(O Retorno???) e sqrt(pi)

2003-03-31 Por tôpico Nicolau C. Saldanha
On Mon, Mar 31, 2003 at 06:58:36PM -0300, João Gilberto Ponciano Pereira wrote:
 É um problema engraçado... Intuitivamente, parece que não dá. Vamos chamar
 de perímetro convexo a soma dos arcos convexos de cada pedaço recortado, e
 perímetro côncavo a soma dos arcos côncavos de cada pedaço recortado.
 
 A figura inicial tem um perímetro convexo igual a 2pi*r, e um perímetro
 côncavo igual a zero. Cada corte em arco, aumenta o perímetro côncavo e o
 perímetro convexo na mesma quantidade. Analogamente, cada colagem, reduzem
 os perímetros da mesma forma. A figura final tem os dois perímetros igual a
 zero...
 
 Tem alguma coisa errada nisso?

Não. O que está errado é o enunciado do Peter Dirichlet.
Veja minha outra mensagem.
 
  Turma,alguem sabe demonstrar esse teorema estranho que me apareceu na
 Semana
  Olimpica?
  Mostre que e possivel recortar um circulo em varios mas finitos pedaços
  e rearranjar os pedaços sem falhas de modo a formar um quadrado.Cada corte
  deve ser ou um arco de circulo ou um segmento de reta.
=
Instruções para entrar na lista, sair da lista e usar a lista em
http://www.mat.puc-rio.br/~nicolau/olimp/obm-l.html
O administrador desta lista é [EMAIL PROTECTED]
=


Re: [obm-l] Problema da Tesoura(O Retorno???) e sqrt(pi)

2003-03-31 Por tôpico Nicolau C. Saldanha
On Mon, Mar 31, 2003 at 03:07:34PM -0300, [EMAIL PROTECTED] wrote:
 Turma,alguem sabe demonstrar esse teorema estranho que me apareceu na Semana
 Olimpica?
 Mostre que e possivel recortar um circulo em varios mas finitos pedaços
 e rearranjar os pedaços sem falhas de modo a formar um quadrado.Cada corte
 deve ser ou um arco de circulo ou um segmento de reta.
 Que tal se esse fosse pra Eureka!?

Isto me cheira ao problema da quadratura do círculo, versão século XX.
O teorema (que não é fácil) é que é possível cortar um quadrado
em um número finito de peças e juntá-las para formar um disco redondo
de mesma área. Mas as peças são muito complicadas, não é possível
resolver o problema se os cortes forem limitados a curvas bem comportadas.

Isso parece o paradoxo de Banach-Tarski: é possível decompor uma bola
em um número finito de pedaços e juntá-los para formar duas bolas,
cada uma igual à bola original. O teorema mais geral é que se A e B
são dois subconjuntos de R^3 limitados e de interior nào vazio então
é possível recortar A em um número finito de pedaços e juntá-los
para montar B. Note em particular que não existe preservação de volume;
em R^2 existe, não é possível recortar uma bola pequena para montar
uma bola grande.

[]s, N.
=
Instruções para entrar na lista, sair da lista e usar a lista em
http://www.mat.puc-rio.br/~nicolau/olimp/obm-l.html
O administrador desta lista é [EMAIL PROTECTED]
=


[obm-l] Re: [obm-l] Re: [obm-l] Fraçao continua de e

2003-03-31 Por tôpico Nicolau C. Saldanha
On Mon, Mar 31, 2003 at 03:42:21PM -0300, Cláudio (Prática) wrote:
 Você acha a demonstração aqui:
 http://research.microsoft.com/~cohn/Papers/e.pdf
 
  Turma,ces ja viram a fraçao continua de
  e=[2;1,2,1,1,4,1,1,6,1,1,8,...,1,1,2n,...]Comom se demonstra isso?

Muito elegante esta demonstração.

Uma solução mais longa mas talvez mais informativa é provar que
a função tanh(x) é dada pela seguinte fração contínua:

x/(1 + x^2/(3 + x^2/(5 + x^2/(7 + x^2/(9 + x^2/(11 + x^2/(13 +...)))

Existem fórmulas parecidas para outras funções; estes são os aproximantes
de Padé, bem melhores do que os de Taylor para vários fins.

Note aliás que a expansão Padé da tangente é bem mais simples que a de Taylor,
já discutida aqui antes.

O Gugu e eu já demonstramos a fórmula acima uma vez a partir das expansões
de Taylor de senh x e cosh x; é meio braçal mas pode ser feito.

[]s, N.
=
Instruções para entrar na lista, sair da lista e usar a lista em
http://www.mat.puc-rio.br/~nicolau/olimp/obm-l.html
O administrador desta lista é [EMAIL PROTECTED]
=


Re: [obm-l] Mais Problemas em Aberto

2003-03-31 Por tôpico Nicolau C. Saldanha
On Mon, Mar 31, 2003 at 03:13:46PM -0300, Cláudio (Prática) wrote:
 2)Determine todos os primos da forma 101010.101.

O único primo é 101.

Defina h(n) = (100^n - 1)/99. Queremos descobrir para quais valores
de n temos h(n) primo. É fácil provar que a|b implica em h(a)|h(b)
donde basta considerar n primo. O caso n=2 nos dá o primo 101 donde
basta considerar n primo ímpar. Mas definindo g(n) = (10^n - 1)/9
temos g(n)|h(n) para todo n ímpar, como se verifica facilmente.

[]s, N.

=
Instruções para entrar na lista, sair da lista e usar a lista em
http://www.mat.puc-rio.br/~nicolau/olimp/obm-l.html
O administrador desta lista é [EMAIL PROTECTED]
=


Re: [obm-l] Iezzi matemetica

2003-03-31 Por tôpico Nicolau C. Saldanha
On Mon, Mar 31, 2003 at 10:28:46AM -0300, Oswaldo Stanziola wrote:
 Felipe,
 Qual é o seu email?

Eu não sou o Felipe, mas eu li o cabeçário da mensagem dele.
Lá aparece o endereço: [EMAIL PROTECTED]

[]s, N.
=
Instruções para entrar na lista, sair da lista e usar a lista em
http://www.mat.puc-rio.br/~nicolau/olimp/obm-l.html
O administrador desta lista é [EMAIL PROTECTED]
=